Finding eigen values of a 2x2 matrix

Click For Summary
The eigenvalues of the 2x2 matrix (0 1)/(-1 0) are correctly calculated as λ = ±i, indicating complex eigenvalues. This suggests that the system exhibits a rotational behavior, specifically a 90-degree planar rotation. The eigenvectors associated with these eigenvalues correspond to the vectors that are rotated by the matrix. Understanding these eigenvalues is crucial for analyzing the system's dynamics and fixed points. The discussion emphasizes the relevance of eigenvalue analysis in describing system behavior.
andrey21
Messages
475
Reaction score
0
Find the eigenvalues of the following 2x2 matrix:

(0 1)/(-1 0)


Homework Equations





By using the forumla (a-λ)(d-λ) -bc I was able to obtain the following:


λ^(2) + 1 = 0
λ^(2) = -1

λ = ± √ (-1)

Is thos correct? Also what relevance does this have on the fixed points?
 
Physics news on Phys.org
Jamiey1988 said:
Find the eigenvalues of the following 2x2 matrix:

(0 1)/(-1 0)


Homework Equations





By using the forumla (a-λ)(d-λ) -bc I was able to obtain the following:


λ^(2) + 1 = 0
λ^(2) = -1

λ = ± √ (-1)

Is thos correct? Also what relevance does this have on the fixed points?
Yes, they are correct. You can also write them as λ = ±i.

Regarding fixed points, you might be talking about this: For some vectors x1 and x2, Ax1 = ix1, and Ax2 =- ix2, where A is your matrix. i and -i are the eigenvalues and the x vectors are eigenvectors.
 
Thanks for that just needed some confirmation:)

I was just reading that if a system has eigen values of i and -i then this corresponds to a 90 degree planar rotation?

I ask because the question I am answering goes on to say.

Use eigen value analysis to dscribe the behaviour of the system.

Any help would be great
 
Question: A clock's minute hand has length 4 and its hour hand has length 3. What is the distance between the tips at the moment when it is increasing most rapidly?(Putnam Exam Question) Answer: Making assumption that both the hands moves at constant angular velocities, the answer is ## \sqrt{7} .## But don't you think this assumption is somewhat doubtful and wrong?

Similar threads

  • · Replies 2 ·
Replies
2
Views
1K
  • · Replies 11 ·
Replies
11
Views
2K
  • · Replies 7 ·
Replies
7
Views
2K
  • · Replies 10 ·
Replies
10
Views
1K
  • · Replies 10 ·
Replies
10
Views
2K
  • · Replies 1 ·
Replies
1
Views
2K
Replies
4
Views
2K
Replies
2
Views
2K
  • · Replies 5 ·
Replies
5
Views
2K
  • · Replies 10 ·
Replies
10
Views
5K